Casey buys a bracelet. She pays for the bracelet and pays \$0.72$0.72 in sales tax. The sales tax rate is 6\%6%. What is the original price of the bracelet, before tax?

Answers

Answer 1

Answer:

The original price of the  bracelet before tax is $12.

Step-by-step explanation:

Let us assume that the original price of the  bracelet be x.

As given

Casey buys a bracelet.

She pays for the bracelet and pays $0.72 in sales tax .

The sales tax rate is 6% .

6% is written in the decimal form.

[tex]= \frac{6}{100}[/tex]

= 0.06

Than the equation becomes

x × 0.06 = 0.72

0.06x = 0.72

[tex]x = \frac{0.72}{0.06}[/tex]

x = $12

Therefore the original price of the  bracelet before tax is $12.





Related Questions

Help me please lots of points

Answers

Answer:

In your first picture

7) -40

14) 200

Step-by-step explanation:

The symbol in the picture is asking for multiplication. Typically for these, teachers encourage the use of the multiplication button on a calculator, but otherwise you need to know the basic steps of multiplication.

Answer:

7) -40

14) 200

Step-by-step explanation:

What is the answer to a+b=s+r; r

Answers

Answer:      " r  =  a  +  b  -  s "  .

_____________________________________________________

Step-by-step explanation:

_____________________________________________________

Given:  

a + b = s + r ;  Solve for:  "r" ;  that is; rewrite the equation in terms of:  " r " .

_____________________________________________________

Subtract "s" from each side of the equation;  

   to isolate "r" on one side of the equation:

_____________________________________________________

    →      a + b - s  =  s + r - s  ;

    →   to get:

   →       a + b - s  =  r  ;

____________________________________________________

    " r = a + b - s "  .

____________________________________________________

   →  which is the answer.

____________________________________________________

Hope this helps!

 Best wishes to you!

____________________________________________________

There are 64 pretzels in a 16-ounce bag of chocolate covered pretezel. What is the rate of pretzels per ounce?

Answers

the answer is 4 pretzels

Ali caught a fish that weighted 9 and 1/2 lb. Lisa caught a fish that weighed 5 and 1/ 3 lb. How much more did ali fish weight.

Answers

Final answer:

Ali's fish weighs 25/6 lb more than Lisa's fish.

Explanation:

To find out how much more Ali's fish weighs compared to Lisa's fish, we need to subtract the weight of Lisa's fish from the weight of Ali's fish.

Ali's fish weighs 9 and 1/2 lb, which can be rewritten as 19/2 lb. Lisa's fish weighs 5 and 1/3 lb, which can be rewritten as 16/3 lb.

Now, subtracting the weight of Lisa's fish from Ali's fish, we have:

Ali's fish weight - Lisa's fish weight = (19/2) lb - (16/3) lb = (57/6) lb - (32/6) lb = 25/6 lb.

Therefore, Ali's fish weighs 25/6 lb more than Lisa's fish.

Justin's rice ball recipe uses 100 grams of rice to make 11 rice ball. Justin has 700grams of rice. How many rice balls can Justin make with 700 grams of rice?

Answers

Answer: I TINK 77 rice balls


Step-by-step explanation:


Answer:77

Step-by-step explanation:

If U = {natural numbers less than 20} and N = {factors of 18}, what is N

Answers

U={0,1,2,3, 4,5,6,7,8,9,10,11,12,13,14,15,16,17,18,19}
N={1,2,3,6,9,18}

This is from the answer key to this question;

N= {4, 5, 7, 8, 10, 11, 12, 13, 14, 15, 16, 17, 19.}

25 POINTS! EXPLAIN FOR BRAINLIEST/THANK YOU.

Answers

By the polynomial remainder theorem, a polynomial [tex]p(x)[/tex] has a factor of [tex]x-c[/tex] if [tex]p(c)=0[/tex]. So all you need to do is check the value of [tex]f(x)[/tex] at [tex]x=1,3,-3,5,-5[/tex]. You should get

[tex]f(1)=-96[/tex] (so [tex]x-1[/tex] is NOT a factor)

[tex]f(3)=-96[/tex] (so [tex]x-3[/tex] is NOT a factor)

[tex]f(-3)=f(5)=f(-5)=0[/tex] (so the last three options are factors)

Answer:

the answer is c and yeah

Step-by-step explanation:

Ms.Rochelle wants to put her 29 students into groups of 6. How many groups of 6 can she make? If she puts any remaining students in a smaller group, how many students will be in that group?

Answers

Answer:

A. 4 groups of 6 students in each group.

B. The smaller group will have 5 students.        

Step-by-step explanation:  

We have been given that Ms.Rochelle wants to put her 29 students into groups of 6.  

A. To find the number of groups of 6 that can be made from 29 students, we will find the greatest multiple of 6 from 29.    

Multiples of 6 are: 6, 12, 18, 24, 30, 36,...  

We can see that the greatest multiples of 6 in 29 in 24.  

[tex]\text{Number of groups that can be made with 29 students}=\frac{24}{6}[/tex]          

[tex]\text{Number of groups that can be made with 29 students}=4[/tex]

Therefore, Ms. Rochelle can form 4 groups from 29 students with 6 students in each group.            

B. To find the number of students in small group, we will subtract 24 from 29.  

[tex]\text{Number of students in the small group}=29-24[/tex]

[tex]\text{Number of students in the small group}=5[/tex]  

Therefore, If Ms. Rochelle puts any remaining students in a smaller group, the number of students in smaller group will be 5.  


HELPP! Calculate S22 for the arithmetic sequence in which a12=2.4 and the common difference is d=3.4

Answers

Answer:

S22 for the arithmetic sequence is:

First option: 15.4

Step-by-step explanation:

a12=2.4

d=3.4

S22=?

Sn=(a1+an)n/2

n=22

S22=(a1+a22)22/2

S22=(a1+a22)11


ak=aj+(k-j)d

a12=a1+(12-1)d

2.4=a1+11(3.4)

2.4=a1+37.4

Solving for a1: Subtracting 37.4 both sides of the equation:

2.4-37.4=a1+37.4-37.4

Subtracting:

-35=a1

a1=-35


a22=a12+(22-12)d

a22=2.4+10(3.4)

a22=2.4+34

a22=36.4


S22=(a1+a22)11

S22=(-35+36.4)11

S22=(1.4)11

S22=15.4

Use the dot product to find |v| when v=(-8,2)

Answers

Answer:

2√17

Step-by-step explanation:

V = (-8, 2)

|v| = √((-8)^2 + 2^2)

= √(64 + 4)

= √68

= √4* √17

= 2√17

Answer:

  = 2 sqrt(17)

Step-by-step explanation:

To find the magnitude of v using the dot product

|v| = sqrt(v1*v1 + v2*v2)

    = sqrt( -8*-8 + 2*2)

    = sqrt(64+ 4)

   = sqrt(68)

   = sqrt(4*17)

  = sqrt(4) sqrt(17)

  = 2 sqrt(17)

What is an equation of the line that is parallel to y=4x-1 and that passes through the point (-3,5)?

A. y=4x-3
B. y=4x+5
C. y=4x+17
D. y=-4x+17

Answers

Answer:

C. y=4x+17

Step-by-step explanation:

If we want to find a line parallel to y =4x-1, the slopes will have to be the same because parallel lines have the same slope.  Knowing y= mx+b we know the slope of the  old line is 4, so the slope of the parallel line is 4.

If we know the slope of the line and a point, we can use the point slope form of a line

y-y1 = m(x-x1)

y-5 = 4(x--3)

y-5=4(x+3)

Distribute the 4

y-5 = 4x+12

Add 5 to each side

y-5+5 = 4x+12+5

y = 4x+17

Answer: The answer is C. y=4x+17

Step-by-step explanation:

The other person was correct and i got an A<3

How do the graphs of these function compare to one another?

Answers

If you’re allowed to, try putting these functions into swamps. It is a free graphing website.

HELP!!!!!! I believe its B but I am not sure! :(

Answers

Answer:

A

Step-by-step explanation:

When solving for x as an exponent, we need to use logarithms in order to undo the operation and rearrange the terms. We use log rules to bring down the exponent and solve. Logarithms are the inverse operations to exponents and vice versa. We have one special kind of logarithm called the natural logarithm whose base is e. We write it as ln. Since our base is e here, we will use the natural logarithm to rearrange and isolate x.

[tex]e^{4x-1} =3[/tex]

We begin by applying the natural logarithm to each side.

[tex]ln(e^{4x-1}) =ln(3)[/tex]

Log rules allow use to rearrange the exponent as multiplication in front of the log.

[tex](4x-1)ln(e) =ln(3)[/tex]

ln e as an inverse simplifies to 1.

[tex](4x-1)(1)=ln(3)[/tex]

We now apply the inverse operations for subtraction and multiplication.

[tex]4x-1+1=1+ln(3)\\4x=1+ln(3)\\\frac{4x}{4} =\frac{1+ln3}{4} \\x =\frac{1+ln3}{4}[/tex]

Option A is correct.

how do you do this? solve for b1

Answers

[tex]\dfrac{1}{2}h(b_1+b_2)=A\qquad\text{multiply both sides by 2}\\\\h(b_1+b_2)=2A\qquad\text{divide both sides by }\ h\neq0\\\\b_1+b_2=\dfrac{2A}{h}\qquad\text{subtract}\ b_2\ \text{from both sides}\\\\\boxed{b_1=\dfrac{2A}{h}-b_2}\to\boxed{F.}[/tex]

Find the next two terms of the sequence: 80, ?40, 20, ?10, ______, ______. A) 20, 40 B) 1 2 , 1 4 C) 5, ? 5 2 D) ?30, ?40

Answers

Answer:

5, 5/2

Step-by-step explanation:

80, 40, 20, 10, ______,

80/40 = 2

40/20 = 2

We are dividing by 2 each time

10/2 =5  so the next term is 5

5/2 = 5/2

Consider the scatter plot.


Scatter plot with line of best fit of y equals 0.75x plus five.



The line of best fit is y = 0.75x + 5.



Choose the best representation for the slope.



The slope of the line of best fit shows that each additional minute, the distance increases by 0.75 feet.

The slope of the line of best fit shows that each additional minute, the distance decreases by 5 feet.

The slope of the line of best fit shows that each additional minute, the distance decreases by 0.75 feet.

The slope of the line of best fit shows that each additional minute, the distance increases by 5 feet.

Answers

Answer:

The correct answer option is: The y-intercept of the line of best fit shows that when time started, the distance was 5 feet.

Step-by-step explanation:

We are given a scatter plot with a best fit line as shown on the given graph.

The equation of the best fit line is given by:

y = 0.75x + 5

So with the help of the equation and by looking at the given graph, we can conclude about the representation of the y intercept that the the y-intercept of the line of best fit shows that when time started, the distance was 5 feet.

Since the distance shown on the y axis is already 5 when the time started at 0 minutes.

List the first four terms of a geometric sequence with t 1 = 4 and t n = -3t n-1 .

Answers

[tex]t_1=4\\t_n=-3t_{n-1}\\\\t_2=-3\cdot4=-12\\t_3=-3\cdot(-12)=36\\t_4=-3\cdot36=-108[/tex]

Algebraic Expressions Write each phrase as an algebraic expression. 5 subtracted from y

Answers

y-5 because 5 is subtracted from y it goes second

Someone help me please 30 points

Answers

Answer:4.-37 5.-6 6.-33 11.-20 12.-16

13.19


Step-by-step explanation:


Answer:

1. -41

2. -40

3. -4

4. -37

5. -24

6. -33

8. -47

9. 14

10. 59

11. -20

12. -16

13. 19


Kris has a box of 8 crayons. Silvia, s box has 6 times of many crayons as Kris box . How many crayons are in sylvia box

Answers

Sylvia would have 48 crayons.
6•8=48

In triangle LNP, R is the centroid and LO = 30. Find LR and RO. Enter the answers as numbers.

Answers

Answer:

[tex]|LR|=20[/tex]

[tex]|RO|=10[/tex]

Step-by-step explanation:

The centroid divides the median in the ratio [tex]2:1[/tex].


Thus [tex]|LR|:|RO|=2:1[/tex]

This implies that,

[tex]|LO|:|RO|=3:1[/tex]

We were given that,

[tex]|LO|=30[/tex]

We substitute this value to obtain,

[tex]30:|RO|=3:1[/tex]


or


[tex]\frac{30}{|RO|}=\frac{3}{1}[/tex]

We cross multiply to get,

[tex]3|RO|=30\times 1[/tex]


We divide through by 3, to get,


[tex]|RO|=10[/tex]



We can observe from the diagram that

[tex]|LR|+|RO|=|LO|[/tex]


[tex]|LR|+10=30[/tex]


[tex]|LR|=30-10[/tex]


[tex]|LR|=20[/tex]




Which of the following points is a solution of the inequality y < -|x|?

A) (1, -2)
B) (1, -1)
C) (1, 0)

Answers

Answer: choice A only

(1,-2) is the only solution (from the list of choices)

====================================

Explanation:

Let's go through each answer choice. We'll plug the coordinates in one at a time.

-------------

Choice A has the point (1,-2) so x = 1 and y = -2 pair up together

y < -|x|

-2 < -|1|

-2 < -1

This is a true statement as -2 is to the left of -1 on the number line. So (1,-2) is one solution. Let's see if there are others.

-------------

Choice B) plug in (x,y) = (1,-1)

y < -|x|

-1 < -|1|

-1 < -1

False. A number is not smaller than itself. So we can cross B off the list.

-------------

Choice C) plug in (x,y) = (1,0)

y < -|x|

0 < -|1|

0 < -1

This is false because -1 is smaller than 0. Cross choice C off the list.

-------------

Only choice A is a solution point for this inequality. If we were to graph the inequality, we would see only point A is in the shaded region while the other points are outside the shaded region.

The dashed boundary line does not count as the shaded region. This visually confirms why point B does not work.

Final answer:

After evaluating each point against the inequality y < -|x|, only point A (1, -2) satisfies the condition, making it the correct answer.

Explanation:

To determine which point is a solution for the inequality y < -|x|, we need to check if the y-value of each point is less than the negative absolute value of its corresponding x-value.

For point A (1, -2), we check if -2 < -|1|. Since |-1| = 1, we are checking if -2 < -1, which is true. So, point A is a solution.For point B (1, -1), we check if -1 < -|1|. Since |-1| = 1, we are verifying if -1 < -1, which is not true. Therefore, point B is not a solution.For point C (1, 0), we check if 0 < -|1|. Since |-1| = 1, we are verifying if 0 < -1, which is not true. Hence, point C is not a solution.

Therefore, the correct answer is point A (1, -2), as it is the only point where the y-value is less than the negative absolute value of the x-value.

Which equation is in point-slope form for the given point and slope?

Answers

Answer: y-7 = 4(x+3), choice B

============================================

Point slope form is generally

y-y1 = m(x-x1)

we have m as the slope and (x1,y1) as the point this line goes through. In this case,

m = 4

(x1,y1) = (-3,7) so x1 = -3 and y1 = 7

So,

y-y1 = m(x-x1)

y - 7 = 4(x - (-3))

y - 7 = 4(x + 3)

which is why choice B is the answer

Point-slope form:

y - y₁ = m(x - x₁)    "m" is the slope


Since you know:

m = 4

(x₁ , y₁) = (-3,7)

You can plug it into the equation


y - y₁ = m(x - x₁)  

y - 7 = 4(x - (-3))

y - 7 = 4(x + 3)

Your answer is the 2nd choice

Jim drank 2/5 of his water bottle and John drank 3/10 of his water bottle. How much did the both boys drink?

Answers

The boys drink 7/10 of a water bottle

1. Convert 2/5 into 4/10
2. Add 4/10 and 3/10

What is the growth factor of the following example? Assume time is measured in the units given.
Water usage is increasing by 3% per year.

Answers

Answer:

The growth factor is option c. 1.03 per year

Step-by-step explanation:

Growth rate: r = 3% = 3 / 100 → r = 0.03

Growth factor: b = 1+r

Replacing r by 0.03 in the formula above:

b = 1+0.03

b = 1.03

Answer: The growth factor is 1.03 per year

The value of x is 12.

Answers

false i think because if it's 12 the angle of triangle should be 60°

Answer: False

Step-by-step explanation:

45-45-90 is a special triangle because there is a specific relationship between the legs and the hypotenuse: a - a - a√2

So, If the leg of the 45-45-90 triangle is 12, then the hypotenuse (labeled as x in your drawing) will be 12√2.

On Friday, 537 people attended a play. On Saturday, 812 people attended the same play. About how many more people attended the play on Saturday than on Friday? Hiw did u estimate? Show your work

Answers

Answer:

275

Step-by-step explanation:

To find out how many more, we find the difference between the two values. We use the math operation subtraction to find the difference.

Friday had 537 and Saturday had 812. We subtract 812-537= 275.

275 more people attended Saturday than Friday.

Need help fast please

Answers

Answer:

13. The slope for a line that is perpendicular to the line y=4x+8 is  -1/4

14. The slope for a line that is perpendicular to the line x=-6 is 0

15. The equation for a line that is perpendicular to the line 8x-4y=12 and passes through the origin is y = - (1/2) x

16. The equation for a line that is perpendicular to the line y=-(1/3)x and passes through the point (0,-10) is y=3x-10

Step-by-step explanation:

13. What is the slope for a line that is perpendicular to the line y=4x+8?

y=mx+b

Comparing with the form slope-intercept, the slope of the given line is the coefficient of x, then the slope of the given line is 4.

A line perpendicular to y=4x+8 must have a slope opposite and inverse, then:

Slope of the perpendicular = - 1/4


14. What is the slope for a line that is perpendicular to the line x=-6?

The line x=-6 is a vertical line. A line perpendicular to the line x=-6 must be a horizontal line (Angle=0°), then:

Slope of the perpendicular = tan Angle = tan 0° = 0


15. Write the equation for a line that is perpendicular to the line 8x-4y=12 and passes through the origin.

8x-4y=12

Isolating y: Subtracting 8x both sides of the equation:

8x-4y-8x=12-8x

-4y=-8x+12

Dividing all the terms by -4:

-4y/(-4)= -8x/(-4)+12/(-4)

y=2x-3

The slope of the given line is 2

The slope of the perpendicular is m=-1/2 (opposite and inverse to the slope of the given line)

The perpendicular passes through the origin:

P1=(0,0)=(x1,y1)→x1=0, y1=0

Using the equation point - slope:

y-y1=m(x-x1)

Replacing the known values:

y-0=(-1/2)(x-0)

y=(-1/2)x


16. Write an equation for a line that is perpendicular to the line y=-(1/3)x and passes through the point (0,-10).

The slope of the given line is -(1/3)

The slope of the perpendicular is m=3/1→m=3 (opposite and inverse to the slope of the given line)

The perpendicular passes through the point:

P1=(0,-10)=(x1,y1)→x1=0, y1=-10

Using the equation point - slope:

y-y1=m(x-x1)

Replacing the known values:

y-(-10)=3(x-0)

y+10=3x

Isolating y: Subtracting 10 both sides of the equation:

y+10-10=3x-10

y=3x-10


 

Solve 14.2 divided by 0.5 dhow your work and explain how you new where thr decamal point whas going to be

Answers

Final answer:

To solve the division problem 14.2 divided by 0.5, you can use the long division method. The quotient is 28.4.

Explanation:

To solve the division problem 14.2 divided by 0.5, we can perform the long division method. Here's the step-by-step process:

Place the dividend, 14.2, inside the division bar.Place the divisor, 0.5, outside the division bar.Divide the first digit of the dividend, which is 1, by the divisor. The result is 2.Write down the quotient, 2, above the division bar.Multiply the quotient, 2, by the divisor, 0.5, which gives you 1.Subtract the product, 1, from the first digit of the dividend, 1, which gives you a remainder of 0.Bring down the next digit, which is 4, from the dividend.Divide the new dividend, 4, by the divisor, 0.5. The result is 8.Write down the new quotient, 8, next to the previous quotient.Multiply the new quotient, 8, by the divisor, 0.5, which gives you 4.Subtract the product, 4, from the new dividend, 4, which gives you a remainder of 0.

Since there is no remainder anymore, the division is complete. The final quotient is 28.4. Therefore, 14.2 divided by 0.5 is equal to 28.4.

A rectangular field is 115 yards long and 75 yards wide.
Give the length and width of another rectangular field that has the same perimeter but a smaller area

Answers

Final answer:

To create a rectangular field with the same perimeter but a smaller area than the original 115 yards by 75 yards field, one could have a field that is 130 yards long and 60 yards wide.

Explanation:

To find the length and width of another rectangular field that has the same perimeter as a 115 yards by 75 yards field but a smaller area, we first calculate the perimeter of the original field:

Perimeter = 2(length + width) = 2(115 yd + 75 yd) = 2(190 yd) = 380 yd

To have a smaller area, the new field cannot be a square (which would maximize the area) and the sides need to have a greater difference in their measurements while still adding up to half of the original perimeter:

Let's assume the new length is 130 yards, to find the new width: 380 yd / 2 - 130 yd = 60 yd

Therefore, a new rectangular field could be 130 yards long and 60 yards wide.

Other Questions
Will give Brainlist! 17 pt HELP PLZ! What should you do if you are on a narrow, two way residential, street and there is not room for your car and another oncoming vehicle to pass each other because cars parked on the shoulders are blocking your path i need help thx so much two angles in a triangle mesaure 65 degrees and 75 degrees what is the measure of the third angle of the triangle A distance of 150 km was covered by a motorcyclist traveling at an average speed of 75 km/h, by a bus at 60 km/h, a truck at 50 km/h, and a bicyclist at 20 km/h. How much time did each require to travel the entire distance? Explain why the speed and the time needed to travel 150 km are inversely proportional quantities. for The motorcyclist required hours, the bus required hours, the truck required hours, the bicyclist required hours. What was the significance of Operation Barbarossa, which began in 1941? A: It allied the Soviet Union with Germany. B: It opened a second front in Eastern Europe. C: It brought the United States into World War II. D: It pulled the defeated French out of the war. A caterer makes 3 extra sandwiches for every 20 sandwiches for a customer order. Write the ratio of ordered sandwiches to extra sandwiches. The caterer makes a total of 184 sandwiches for a customer. How many sandwiches did the customer order? Pressure from the Arabian Plate pressing against the Eurasian Plate created a mountains in Iraq and Oman. b fold traps in the African Plate. (wrong) c the Tigris and Euphrates Rivers. (also wrong) d a broad valley along the shores of the Red Sea. General conic equation i need help with these problems please show your work how do we measure or think of speed in the united states? Which does not usually trigger mass movements? vibration of the ground during an earthquake formation of oversteepened slopes growth of native vegetation on slopes saturation of surface materials with water What similarities are there between the land ordinance of 1785 and the trail of tears Need answer ASAP! I only have two questions left!!!What is the name of an interior angle of a triangle that shares a side and a vertex with an exterior angle of the triangle? A) exterior angleB) adjacent interior angleC) remote interior angleD) alternate interior angle HELP!!!!!!!!!!!!!!!!!!!!!!!!!!!!!!!!!!!!!!!!!!!!!! Arlene has developed a disorder in which she occasionally stops breathing during her sleep this is called: using the polygon angle sum theorem, find x in the graphic aboveA) 720B) 80C)100D)120 JJ decides to leave a tip that is 15% of the original price for his meal. How much should he leave as tip for the $5.50 cheeseburger and $2.50 milkshake? who were the janissaries Approximately how many moons does Saturn have Wild crickets living in a field have a poplulation density of 2.4 crickets per square meter. If the fields area is 250 per square meter, what is the approximate size of the cricket population? Steam Workshop Downloader